1
$\begingroup$

I have a problem in which I have two (though there can be more) multivariate Gaussian functions very close to each other and I want to find whether being next together causes the sum of these two Gaussian functions to be unimodal or bimodal (more than one peak). I have tried to solve this problem analytically so far; since the type of multivariate Gaussians I am using looks something like this:

$$f(x)=\frac{1}{\sqrt{(2\pi)^k|\Sigma|}}\exp\left(-\frac{1}{2}(x-m)^T\Sigma^{-1}(x-m)\right)$$

where x is a k dimensional vector and sigma is the k x k covariance matrix. The obvious thing for me to do was the take the derivative since I am trying to find the peaks, which gives me something like this: $$\frac{\partial f(x)}{\partial x}=f(x)\Sigma^{-1}(x-m)$$

Now I'm at a loss since I need to solve this equation for all the zeros and I don't know how to do that. I know that there are optimising solutions out there to find the solutions and since my initial guess would be near m so that could work well except I am trying to figure out if the derivative has more than one peak and where it is - that method only tells me where one is; therefore, I need a method to find all zeros/roots. Does anyone know how to go about solving either the initial problem statement or finding the roots of the derivative. (I work in Python, but since the question is so open ended any solution would be appreciated)

$\endgroup$

1 Answer 1

1
$\begingroup$

Unfortunately, this problem seems to be very hard in the general case: for instance, your distribution might have 3 modes (hence something like 6 zeroes of the gradient, including saddle points), even if there are only 2 mixture components! There's no closed-form formula for the modes either.

If the distributions are sufficiently close together compared to their variances, you might be able to get away with starting a gradient optimization at the mean of each individual distribution and calling the mixture "unimodal" if they converge to the same result. But I'm not sure if this will always work.

There's been some discussion of a more general version of this question here.

$\endgroup$

Your Answer

By clicking “Post Your Answer”, you agree to our terms of service and acknowledge you have read our privacy policy.

Not the answer you're looking for? Browse other questions tagged or ask your own question.